Электроны, летящие в телевизионной трубке, обладают энергией 12 кэВ. Трубка ориентирована так, что электроны движутся горизонтально с юга на север. Вертикальная составляющая земного магнитного поля направлена вниз, и его индукция В=5,5 . 10^-5 Тл. В каком направлении будет отклоняться электронный луч? Каково ускорение каждого электрона? На сколько отклонится пучок электронов, пролетев 20 см внутри телевизионной трубки?

Сила Лоренца F перпендикулярна векторам В и v , и её направление определяется с помощью того же правила левой руки, что и направление силы Ампера: если левую руку расположить так, чтобы составляющая магнитной индукции В, перпендикулярная скорости заряда, входила в ладонь, а четыре пальца были направлены по движению положительного заряда (против движения отрицательного), то отогнутый на 90 градусов большой палец покажет направление действующей на заряд силы Лоренца F л. 

Как видим, электрон под воздействием магнитного поля за счет силы Лоренца будет  отклоняться от прямолинейной траектории с юга на север в сторону востока. 

Сила Лоренца:   $F=qvB\sin\alpha$      (1)

Заряд электрона q = 1.60217657 × 10-19 Кулона

a = 90,    sin a = 1   

 Скорость найдем, зная энергию.  

Кинетическая энергия:        $E=\frac{mv^2}{2}$        

Откуда скорость:   $v=\sqrt{\frac{2E}{m}}$           (2)

Подставим все в (1):                  $F=qB\sqrt{\frac{2E}{m}}$     

Ускорение находим по второму закону Ньютона:       

$a=\frac{F}{m}=\frac{qB\sqrt{\frac{2E}{m}}}{m}$         (3)

Это центростремительное ускорение, направленное в любой момент времени перпендикулярно к скорости электрона. 

$a=\frac{1,6*10^{-19}*5,5*10^{-5}\sqrt{\frac{2*1,923*10^{-15}}{9,1*10^{-31}}}}{9,1*10^{-31}}\approx 6,3*10^{14}\;\text{м/с}^2$

Найдем время пролета электрона внутри трубки t, учитывая, что найденная нами в (2) его начальная скорость  была направлена вдоль оси х (с юга на север).  Эта скорость на рисунке показана, как Vx.  
$t=\frac{x}{v}$  

Тогда, для равноускоренного движения в направлении востока найдем отклонение за время t:

$d=\frac{at^2}{2}$           

 $d=\frac{\frac{qBx^2\sqrt{\frac{2E}{m}}}{m}}{2*\frac{2E}{m}}=\frac{qBx^2\sqrt{\frac{2E}{m}}}{4E}$

Комментарии